Suppose


(a) Find and .




(b) Sketch the phase plane trajectory that satisfies the given initial condition. Which graph most closely resembles the graph you drew?

(c) Is the solution curve headed toward or away from the origin as increases?




 
 
A   B
 
C   D

In order to get credit for this problem all answers must be correct.